WBR1032

Revision as of 00:01, 21 January 2014 by Vendhan Ramanujam (talk | contribs) (Created page with "{{WBRQuestion |QuestionAuthor=Vendhan Ramanujam |ExamType=USMLE Step 3 |MainCategory=Community Medical Health Center, Primary Care Office |SubCategory=Musculoskeletal/Rheumato...")
(diff) ← Older revision | Latest revision (diff) | Newer revision → (diff)
Jump to navigation Jump to search
 
Author PageAuthor::Vendhan Ramanujam
Exam Type ExamType::USMLE Step 3
Main Category MainCategory::Community Medical Health Center, MainCategory::Primary Care Office
Sub Category SubCategory::Musculoskeletal/Rheumatology
Prompt [[Prompt::A 74 year old woman with complains of weight loss, progressive malaise, jaw claudication and scalp tenderness for the past 1 month. She also complains of one episode of transient complete loss of vision in her right eye and partial loss of vision in her left eye. On examination, she has a right tender, nodular and non-pulsatile temporal artery. Her lab tests revealed an erythrocyte sedimentation rate of 74 mm/hour and C-reactive protein of 52 mg/liter. She is immediately started on intravenous methylprednisolone 250 mg BD and oral prednisolone 80 mg OD. A following temporal artery biopsy is consistent with arteritis. Her symptoms resolved and her inflammatory markers improved over the next 3 days. She is discharged on oral prednisolone. Three months later she presents to the office with complaints of weakness. She has difficulty in standing up from sitting position and in climbing the stairs. Her lab tests revealed erythrocyte sedimentation rate of 10 mm/hour, C-reactive protein of 3 mg/liter and creatinine phosphokinase of 24 U/L. What is the most likely cause for this patient’s complaints?]]
Answer A AnswerA::Polymyalgia rheumatica
Answer A Explanation AnswerAExp::'''Incorrect'''-
Answer B AnswerB::Inflammatory myositis
Answer B Explanation AnswerBExp::'''Incorrect'''-
Answer C AnswerC::Drug induced myopathy
Answer C Explanation AnswerCExp::'''Correct'''-
Answer D AnswerD::Chronic fatigue syndrome
Answer D Explanation AnswerDExp::'''Incorrect'''-
Answer E AnswerE::Fibromyalgia
Answer E Explanation AnswerEExp::'''Incorrect'''-
Right Answer RightAnswer::C
Explanation [[Explanation::

Educational Objective:
References: ]]

Approved Approved::Yes
Keyword WBRKeyword::Drug induced myopathy, WBRKeyword::Steroid induced myopathy
Linked Question Linked::
Order in Linked Questions LinkedOrder::